Mexican Math Olympiad Problem.

Let n n be a natural number and 1 = d 1 < d 2 < . . . < d k = n 1 = { d }_{ 1 } < {d}_{2} < ... < {d}_{k} = n its divisors, with k 3 k \ge 3 . Find the sum of all n n such that n = d 2 2 + d 3 3 n = d_2^2 + d_3^3 .


The answer is 68.

This section requires Javascript.
You are seeing this because something didn't load right. We suggest you, (a) try refreshing the page, (b) enabling javascript if it is disabled on your browser and, finally, (c) loading the non-javascript version of this page . We're sorry about the hassle.

3 solutions

Kazem Sepehrinia
Jul 18, 2015

Suppose n n is an odd number, then all of it's divisors will be odd and n = d 2 2 + d 3 2 n=d_2^2+d_3^2 will be even. Contradiction, n n is even. It's immediate that d 2 = 2 d_2=2 and n = 4 + d 3 3 n=4+d_3^3 Now d 3 n d_3|n and d 3 d 3 3 d_3|d_3^3 , so we must have d 3 4 d_3|4 and d 3 = 4 d_3=4 since d 3 > 2 d_3>2 . n = 2 2 + 4 3 = 68 n=2^2+4^3=68 n = 68 n=68 indeed satisfies the conditions of problem.

Billy Sugiarto
Jul 22, 2015

Let n = p 1 a 1 p 2 a 2 p 3 a 3 . . . p k a k n = p_{1}^{a_{1}} p_{2}^{a_{2}} p_{3}^{a_{3}} ...p_{k}^{a_{k}} for a positive integer k k and a k N k N a_{k} \in N \forall k \in N .

It is obvious that d 2 = p 1 d_{2} = p_{1} .

While d 3 d_{3} can have two forms which is : d 3 = p 2 d_{3} = p_{2} if and only if p 1 2 > p 2 p_{1}^{2} > p_{2} . That is if p 1 2 n p_{1}^{2} | n . Otherwise d 3 = p 1 2 d_{3} = p_{1}^{2} .

(i) d 3 = p 2 d_{3} = p_{2}

We have n = p 1 2 + p 2 3 n = p_{1}^{2} + p_{2}^{3} . Let p 1 = 2 p_{1} = 2 , 2 n 2 | n .

Therefore n = p 2 3 + 4 1 , 1 ( m o d 4 ) n = p_{2}^{3} + 4 \equiv -1, 1 (mod 4) as p 2 3 p_{2} \geq 3 . Thus 2 n 2 \nmid n . Contradiction.

Now let p 1 3 p_{1} \geq 3 , thus 2 n 2 \nmid n .

We have, n = p 2 3 + 1 0 , 2 ( m o d 4 ) n = p_{2}^{3} + 1 \equiv 0, 2 (mod 4) . Thus 2 n 2 | n . Contradiction.

(ii) d 3 = p 1 2 d_{3} = p_{1}^{2}

Therefore, n = p 1 2 + p 1 6 n = p_{1}^{2} + p_{1}^{6}

Let p 1 3 p_{1} \geq 3 . Thus 2 n 2 \nmid n . But n = p 1 2 + p 1 6 0 ( m o d 2 ) n = p_{1}^{2} + p_{1}^{6} \equiv 0 (mod 2) . Contradiction.

Let p 1 = 2 p_{1} = 2 ,

We have n = 4 + 64 = 68 n = 4 + 64 = 68 with 2 2 < 17 2^{2} < 17 . Therefore n = 68 n = 68 satisfies the above equality.

This is the one I have. Thanks for share your solution!

Gerardo Hernandez - 5 years, 10 months ago
Maggie Miller
Jul 17, 2015

Let n n be such an integer. Let d = ( d 2 , d 3 ) d=(d_2,d_3) . Note that d d 2 d\leq d_2 and d n d|n , so d = 1 d=1 or d = d 2 d=d_2 . Moreover, d 1 ( n d 2 2 ) = d 3 3 d_1|(n-d_2^2)=d_3^3 . Since d 2 , d 3 > 1 d_2,d_3>1 , we conclude d 2 , d 3 d_2,d_3 are not coprime, so d = d 2 d=d_2 . Thus, d 3 = k d 2 d_3=kd_2 for some integer k k .

Then k n k|n and k d 3 k\le d_3 , so k = 1 k=1 or k = d 2 k=d_2 . However, d 2 d 3 d_2\neq d_3 , so k 1 k\neq 1 . Therefore, d 3 = d 2 2 d_3=d_2^2 .

Note that d 2 d_2 is the smallest prime factor of n n . If d 2 d_2 is odd, then n = d 2 2 + d 2 6 n=d_2^2+d_2^6 is even, so 2 n 2|n , yielding a contradiction. Therefore, d 2 d_2 is even, so d 2 = 2 d_2=2 .

Thus, n = 2 2 + 2 6 = 68 n=2^2+2^6=68 . Moreover, 68 68 satisfies the given conditions since its smallest factors greater than 1 1 are indeed 2 2 and 4 4 . Therefore, the answer is 68 \boxed{68} .

0 pending reports

×

Problem Loading...

Note Loading...

Set Loading...